Question and Answers Forum

All Questions      Topic List

Integration Questions

Previous in All Question      Next in All Question      

Previous in Integration      Next in Integration      

Question Number 138801 by mnjuly1970 last updated on 18/Apr/21

          lim_( n→∞) (∫_0 ^( 1) (((1−x)^n e^x )/(n!))dx)=?

$$\: \\ $$$$\:\:\:\:\:\:\:{lim}_{\:{n}\rightarrow\infty} \left(\int_{\mathrm{0}} ^{\:\mathrm{1}} \frac{\left(\mathrm{1}−{x}\right)^{{n}} {e}^{{x}} }{{n}!}{dx}\right)=? \\ $$

Answered by Kamel last updated on 18/Apr/21

Commented by mnjuly1970 last updated on 18/Apr/21

thanks mr kamel   very nice...

$${thanks}\:{mr}\:{kamel}\: \\ $$$${very}\:{nice}... \\ $$

Answered by mr W last updated on 18/Apr/21

∫_0 ^1 0dx≤∫_0 ^( 1) (((1−x)^n e^x )/(n!))dx≤∫_0 ^1 (e^x /(n!))dx  0≤∫_0 ^( 1) (((1−x)^n e^x )/(n!))dx≤((e−1)/(n!))  0≤lim_(n→∞) ∫_0 ^( 1) (((1−x)^n e^x )/(n!))dx≤0  ⇒lim_(n→∞) ∫_0 ^( 1) (((1−x)^n e^x )/(n!))dx=0

$$\int_{\mathrm{0}} ^{\mathrm{1}} \mathrm{0}{dx}\leqslant\int_{\mathrm{0}} ^{\:\mathrm{1}} \frac{\left(\mathrm{1}−{x}\right)^{{n}} {e}^{{x}} }{{n}!}{dx}\leqslant\int_{\mathrm{0}} ^{\mathrm{1}} \frac{{e}^{{x}} }{{n}!}{dx} \\ $$$$\mathrm{0}\leqslant\int_{\mathrm{0}} ^{\:\mathrm{1}} \frac{\left(\mathrm{1}−{x}\right)^{{n}} {e}^{{x}} }{{n}!}{dx}\leqslant\frac{{e}−\mathrm{1}}{{n}!} \\ $$$$\mathrm{0}\leqslant\underset{{n}\rightarrow\infty} {\mathrm{lim}}\int_{\mathrm{0}} ^{\:\mathrm{1}} \frac{\left(\mathrm{1}−{x}\right)^{{n}} {e}^{{x}} }{{n}!}{dx}\leqslant\mathrm{0} \\ $$$$\Rightarrow\underset{{n}\rightarrow\infty} {\mathrm{lim}}\int_{\mathrm{0}} ^{\:\mathrm{1}} \frac{\left(\mathrm{1}−{x}\right)^{{n}} {e}^{{x}} }{{n}!}{dx}=\mathrm{0} \\ $$

Commented by mnjuly1970 last updated on 18/Apr/21

grateful mr W...

$${grateful}\:{mr}\:{W}... \\ $$

Terms of Service

Privacy Policy

Contact: info@tinkutara.com